LSAT and Law School Admissions Forum

Get expert LSAT preparation and law school admissions advice from PowerScore Test Preparation.

 Administrator
PowerScore Staff
  • PowerScore Staff
  • Posts: 8916
  • Joined: Feb 02, 2011
|
#43096
Please post your questions below!
 mscrisbrown
  • Posts: 4
  • Joined: Dec 23, 2018
|
#61542
Hi, can #16 be discussed please.
 Robert Carroll
PowerScore Staff
  • PowerScore Staff
  • Posts: 1787
  • Joined: Dec 06, 2013
|
#61553
Cris,

I'll direct you first to the setup for this: lsat/viewtopic.php?t=16332

Here, we're diagramming this as a Grouping Game, where a station's remaining open means it's "in" the group and a station's being closed means it's not in the group.

The first rule entails that N, R, or both must always be open - so at least one of those is in at all times. In question #16, we have exactly two stations open, so we have to be very careful - the first rule alone is already filling at least one of those two spaces.

The second and third rules take care of the rest. Each rule has, as a sufficient condition, a station from the first rule. Each one says that if that station is open, another station is also open. If we have only two stations open in total, then the first three rules tell us it's some pair of stations from the group {L, M, N, R}. Not all of these pairs work, but the pair of stations open MUST be a pair in this group, because of the consequences of the rules. So, if only two stations are open, nothing outside this group could be open. That includes P and Q. Only Q is an answer, so answer choice (D) is correct for this question.

Robert Carroll

Get the most out of your LSAT Prep Plus subscription.

Analyze and track your performance with our Testing and Analytics Package.